Why do we say ‘pairwise disjoint’, rather than ‘disjoint’?

The name of the pictureThe name of the pictureThe name of the pictureClash Royale CLAN TAG#URR8PPP












32












$begingroup$


I don’t see the ambiguity that ‘pairwise’ resolves.



Surely if $A$, $B$ and $C$ are disjoint sets then they are pairwise disjoint and vice versa?



Or am I being dim?










share|cite|improve this question











$endgroup$







  • 1




    $begingroup$
    Possible duplicate of pairwise disjoint , or disjoint
    $endgroup$
    – YuiTo Cheng
    Mar 9 at 9:35






  • 5




    $begingroup$
    Note to Reviewers: the target post provided by YuiTo Cheng has much less content. Here the question post is poor but there are some good answers demonstrating the lack of consensus on this terminology.
    $endgroup$
    – Lee David Chung Lin
    Mar 10 at 0:24







  • 2




    $begingroup$
    This comment is not directly about sets, but about a similar situation that arises with relative primality of integers. The numbers 6, 10, and 15 are relatively prime as a triple ($gcd(6,10,15) = 1$) but they are not pairwise relatively prime. For positive integers $a_1, ldots, a_n$ where $n geq 3$, there is a distinction between being pairwise relatively prime ($gcd(a_i,a_j) = 1$ for all $i not= j$) and being relatively prime as an $n$-tuple ($gcd(a_1, ldots, a_n) = 1$). If you want to say $n$ positive integers are "relatively prime" and $n geq 3$ then you should be more precise.
    $endgroup$
    – KCd
    Mar 10 at 14:06
















32












$begingroup$


I don’t see the ambiguity that ‘pairwise’ resolves.



Surely if $A$, $B$ and $C$ are disjoint sets then they are pairwise disjoint and vice versa?



Or am I being dim?










share|cite|improve this question











$endgroup$







  • 1




    $begingroup$
    Possible duplicate of pairwise disjoint , or disjoint
    $endgroup$
    – YuiTo Cheng
    Mar 9 at 9:35






  • 5




    $begingroup$
    Note to Reviewers: the target post provided by YuiTo Cheng has much less content. Here the question post is poor but there are some good answers demonstrating the lack of consensus on this terminology.
    $endgroup$
    – Lee David Chung Lin
    Mar 10 at 0:24







  • 2




    $begingroup$
    This comment is not directly about sets, but about a similar situation that arises with relative primality of integers. The numbers 6, 10, and 15 are relatively prime as a triple ($gcd(6,10,15) = 1$) but they are not pairwise relatively prime. For positive integers $a_1, ldots, a_n$ where $n geq 3$, there is a distinction between being pairwise relatively prime ($gcd(a_i,a_j) = 1$ for all $i not= j$) and being relatively prime as an $n$-tuple ($gcd(a_1, ldots, a_n) = 1$). If you want to say $n$ positive integers are "relatively prime" and $n geq 3$ then you should be more precise.
    $endgroup$
    – KCd
    Mar 10 at 14:06














32












32








32


4



$begingroup$


I don’t see the ambiguity that ‘pairwise’ resolves.



Surely if $A$, $B$ and $C$ are disjoint sets then they are pairwise disjoint and vice versa?



Or am I being dim?










share|cite|improve this question











$endgroup$




I don’t see the ambiguity that ‘pairwise’ resolves.



Surely if $A$, $B$ and $C$ are disjoint sets then they are pairwise disjoint and vice versa?



Or am I being dim?







elementary-set-theory terminology definition






share|cite|improve this question















share|cite|improve this question













share|cite|improve this question




share|cite|improve this question








edited Mar 10 at 14:00









Rócherz

3,0263822




3,0263822










asked Mar 8 at 15:48









John Lawrence AspdenJohn Lawrence Aspden

416410




416410







  • 1




    $begingroup$
    Possible duplicate of pairwise disjoint , or disjoint
    $endgroup$
    – YuiTo Cheng
    Mar 9 at 9:35






  • 5




    $begingroup$
    Note to Reviewers: the target post provided by YuiTo Cheng has much less content. Here the question post is poor but there are some good answers demonstrating the lack of consensus on this terminology.
    $endgroup$
    – Lee David Chung Lin
    Mar 10 at 0:24







  • 2




    $begingroup$
    This comment is not directly about sets, but about a similar situation that arises with relative primality of integers. The numbers 6, 10, and 15 are relatively prime as a triple ($gcd(6,10,15) = 1$) but they are not pairwise relatively prime. For positive integers $a_1, ldots, a_n$ where $n geq 3$, there is a distinction between being pairwise relatively prime ($gcd(a_i,a_j) = 1$ for all $i not= j$) and being relatively prime as an $n$-tuple ($gcd(a_1, ldots, a_n) = 1$). If you want to say $n$ positive integers are "relatively prime" and $n geq 3$ then you should be more precise.
    $endgroup$
    – KCd
    Mar 10 at 14:06













  • 1




    $begingroup$
    Possible duplicate of pairwise disjoint , or disjoint
    $endgroup$
    – YuiTo Cheng
    Mar 9 at 9:35






  • 5




    $begingroup$
    Note to Reviewers: the target post provided by YuiTo Cheng has much less content. Here the question post is poor but there are some good answers demonstrating the lack of consensus on this terminology.
    $endgroup$
    – Lee David Chung Lin
    Mar 10 at 0:24







  • 2




    $begingroup$
    This comment is not directly about sets, but about a similar situation that arises with relative primality of integers. The numbers 6, 10, and 15 are relatively prime as a triple ($gcd(6,10,15) = 1$) but they are not pairwise relatively prime. For positive integers $a_1, ldots, a_n$ where $n geq 3$, there is a distinction between being pairwise relatively prime ($gcd(a_i,a_j) = 1$ for all $i not= j$) and being relatively prime as an $n$-tuple ($gcd(a_1, ldots, a_n) = 1$). If you want to say $n$ positive integers are "relatively prime" and $n geq 3$ then you should be more precise.
    $endgroup$
    – KCd
    Mar 10 at 14:06








1




1




$begingroup$
Possible duplicate of pairwise disjoint , or disjoint
$endgroup$
– YuiTo Cheng
Mar 9 at 9:35




$begingroup$
Possible duplicate of pairwise disjoint , or disjoint
$endgroup$
– YuiTo Cheng
Mar 9 at 9:35




5




5




$begingroup$
Note to Reviewers: the target post provided by YuiTo Cheng has much less content. Here the question post is poor but there are some good answers demonstrating the lack of consensus on this terminology.
$endgroup$
– Lee David Chung Lin
Mar 10 at 0:24





$begingroup$
Note to Reviewers: the target post provided by YuiTo Cheng has much less content. Here the question post is poor but there are some good answers demonstrating the lack of consensus on this terminology.
$endgroup$
– Lee David Chung Lin
Mar 10 at 0:24





2




2




$begingroup$
This comment is not directly about sets, but about a similar situation that arises with relative primality of integers. The numbers 6, 10, and 15 are relatively prime as a triple ($gcd(6,10,15) = 1$) but they are not pairwise relatively prime. For positive integers $a_1, ldots, a_n$ where $n geq 3$, there is a distinction between being pairwise relatively prime ($gcd(a_i,a_j) = 1$ for all $i not= j$) and being relatively prime as an $n$-tuple ($gcd(a_1, ldots, a_n) = 1$). If you want to say $n$ positive integers are "relatively prime" and $n geq 3$ then you should be more precise.
$endgroup$
– KCd
Mar 10 at 14:06





$begingroup$
This comment is not directly about sets, but about a similar situation that arises with relative primality of integers. The numbers 6, 10, and 15 are relatively prime as a triple ($gcd(6,10,15) = 1$) but they are not pairwise relatively prime. For positive integers $a_1, ldots, a_n$ where $n geq 3$, there is a distinction between being pairwise relatively prime ($gcd(a_i,a_j) = 1$ for all $i not= j$) and being relatively prime as an $n$-tuple ($gcd(a_1, ldots, a_n) = 1$). If you want to say $n$ positive integers are "relatively prime" and $n geq 3$ then you should be more precise.
$endgroup$
– KCd
Mar 10 at 14:06











9 Answers
9






active

oldest

votes


















21












$begingroup$

The word "pairwise" in "pairwise disjoint" is superfluous: a collection of sets is disjoint if no element appears in more than one of the sets at a time, and this means that every pair of distinct sets in the collection has an empty intersection. However including the "pairwise" emphasizes that the property can be checked at the level of pairs from the collection (unlike for instance linear independence of vectors in linear algebra). A "disjoint union" is a union of pairwise disjoint sets; one does not say "pairwise disjoint union".



To corroborate my point of view, here is a citation from Halmos:




Pairs of sets with an empty intersection occur frequently enough to justify
the use of a special word: if $Acap B=emptyset$, the sets $A$ and $B$ are
called disjoint. The same word is sometimes applied to a collection of sets
to indicate that any two distinct sets of the collection are disjoint;
alternatively we may speak in such a situation of a pairwise disjoint
collection.




By the way, it is amazing how this site contains lots of answers saying disjoint (for any collection of sets) means empty intersection, like at this question and questions linked from there, and lots of comments saying that is wrong. Which was my motivation to post this as an answer.






share|cite|improve this answer











$endgroup$








  • 2




    $begingroup$
    This answer shows another case where the same terminology means something different to different people. To me a disjoint union is a specific formulation of the coproduct in $mathbfSet$ like the definition on Wikpedia rather than a union of pairwise disjoint sets. But this difference is a bit more immaterial because they are isomorphic when the sets involved are pairwise disjoint.
    $endgroup$
    – Potato44
    Mar 9 at 4:48






  • 1




    $begingroup$
    @Potato44: Yes I know, but I did not want to bring up the distinction internal/external disjoint union, which is similar to internal/external direct sum, which has nothing to do with the current question and would be confusing. My point is that nobody says, or should have to say, "pairwise disjoint union".
    $endgroup$
    – Marc van Leeuwen
    Mar 9 at 9:15










  • $begingroup$
    I just checked Royden (4th edition). It uses disjoint to denote what others here call "pairwise disjoint." Same as Karatzas and Shreve, Pugh in his "Real Mathematical Analysis" and Abbott in his "Understanding Analysis." These are the texts I have in a pdf form so I could search quickly. I have never heard or seen anybody calling a collection of sets disjoint to mean empty intersection.
    $endgroup$
    – Mdoc
    Mar 28 at 2:17



















35












$begingroup$

As evidenced by the answers and comments on this page, the term "disjoint" is ambiguous - some use it to mean "pairwise disjoint", others use it to mean "empty intersection".



Thus, for the sake of clarity, I'd recommend avoiding "disjoint" and using "pairwise disjoint".






share|cite|improve this answer









$endgroup$








  • 4




    $begingroup$
    Could you back up the claim that 'others use it to mean "empty intersection"' (in the case of more than two sets) with any mathematical text (not with answers on this site, which are indeed east to find)?
    $endgroup$
    – Marc van Leeuwen
    Mar 9 at 12:30






  • 1




    $begingroup$
    I agree with your conclusion, and with @MarcvanLeeuwen's comment: I don't think I've ever seen "disjoint" alone for more than $2$ sets, and I think it would be really weird.
    $endgroup$
    – Arnaud Mortier
    Mar 10 at 0:39


















33












$begingroup$

$1,2,2,3,1,3$ are disjoint but not pairwise disjoint.






share|cite|improve this answer









$endgroup$








  • 20




    $begingroup$
    Really? Who would call those disjoint sets?
    $endgroup$
    – John Lawrence Aspden
    Mar 8 at 15:52






  • 38




    $begingroup$
    Everyone. Disjoint means their intersection is empty.
    $endgroup$
    – saulspatz
    Mar 8 at 15:53






  • 16




    $begingroup$
    If sets $A_1,A_2,dots,A_n$ are said to be disjoint then usually it is meant that the sets are pairwise disjoint. See Wolfram for instance. In the other case one says simply that the sets have an empty intersection.
    $endgroup$
    – drhab
    Mar 8 at 16:16







  • 22




    $begingroup$
    So be it. But actually I am the living proof that "everyone" in your first comment is not correct. Lots of times I heard "them" say things like: "we have $mu(bigcup_n=1^inftyA_n)=sum_n=1^inftymu(A_n)$ if the sets are disjoint" (so leaving out pairwise). I do that myself too. It is a good thing however to have a proper definition of this excluding ambiguities. So let's look at this as abuse of language.
    $endgroup$
    – drhab
    Mar 8 at 16:37







  • 17




    $begingroup$
    These sets are not disjoint, at least not by the modern usage of the word “disjoint,” which afaik is used interchangeably with “pairwise disjoint.”
    $endgroup$
    – Shalop
    Mar 8 at 17:33



















15












$begingroup$

Two sets are disjoint when their intersection is empty. Sets are pairwise disjoint when any two of them are disjoint. Most if not all mathematicians also call such sets disjoint, making pairwise a superfluous term for emphasis.






share|cite|improve this answer











$endgroup$








  • 6




    $begingroup$
    I have never seen this sense of disjoint for more than two sets been defined or used in mathematics. The Wikipedia article for disjoint sets does not mention it, and the notion of "disjoint union" definitely refers to pairwise disjoint sets.
    $endgroup$
    – Marc van Leeuwen
    Mar 8 at 19:45






  • 1




    $begingroup$
    @MarcvanLeeuwen Three-or-more sets that are disjoint-but-not-pairwise-disjoint in the sense I describe probably warrant some name, but as you note it's not usually the nomenclature I mentioned, probably because it makes "pairwise" compulsory more than anyone wants to write it.
    $endgroup$
    – J.G.
    Mar 8 at 19:51







  • 4




    $begingroup$
    @MarcvanLeeuwen From the second sentence in the Wikipedia article: "disjoint sets are sets whose intersection is the empty set." If this definition was solely meant for pairs of sets, I would expect it to say "two sets are disjoint if..." rather than "disjoint sets are...".
    $endgroup$
    – Ryan
    Mar 9 at 4:49






  • 1




    $begingroup$
    @Ryan: Thank you for that comment, it allowed me to find the citation of Halmos that I added to my answer (the one provided in the footnote to your WP quote) and which clearly shows that the WP quote is incorrectly phrased. Elsewhere in the WP article (like in the opening phrase) they are careful to always specify intersection of two sets. I'll correct the WP intro.
    $endgroup$
    – Marc van Leeuwen
    Mar 9 at 7:50






  • 1




    $begingroup$
    @MarcvanLeeuwen I've amended my answer to reflect the usage consensus.
    $endgroup$
    – J.G.
    Mar 9 at 8:17


















8












$begingroup$

Let $A=1,2, B=2,3,C=3,4$. Then the sets are disjoint because $Acap Bcap C=emptyset$, but not pairwise disjoint because you have pairs such as $A,B$ such that $Acap Bnot =emptyset$.






share|cite|improve this answer











$endgroup$












  • $begingroup$
    Rats! What is the notation for an empty set?
    $endgroup$
    – Oscar Lanzi
    Mar 8 at 15:54










  • $begingroup$
    Thank you, @jg.
    $endgroup$
    – Oscar Lanzi
    Mar 8 at 15:58






  • 1




    $begingroup$
    I use varnothing, "$varnothing$".
    $endgroup$
    – Eric Towers
    Mar 9 at 22:14


















8












$begingroup$

This is a question about terminology and its usage in practice, so the basis for an answer should come from real quotations and the historical record rather than unsupported opinion. Looking at early known uses of the term, the few sources that I examined all used "disjoint" in its pairwise sense rather than in the sense of having empty intersection.



The earliest published use (known to me) of the term "disjoint" for its mathematical meaning is in a paper "The Thesis of Modern Logistic" (1909), which is the earliest such use found in a JSTOR search and the earliest such use listed in Earliest Known Uses of
Some of the Words of Mathematics
. (Note: Here and throughout, I do not claim that the quoted sources are truly the earliest sources, but merely the earliest ones I was able to find. It is clear that readily available search tools today cover only a small part of historical record. Especially missing from my research is German papers and books, which surely are a large part of the early history of set theory.)



This first paper uses "disjoint" in its description of a construction similar to what we now call an $n$-ary Cartesian product, but where the members of the product are sets rather than ordered $n$-tuples:




Multiplication of cardinals is also defined in purely logical terms. This is done by means of the concept (due to Whitehead) of multiplicative class, which is itself given in terms of logical constants: $k$ being a class of disjoint classes, the multiplicative class of $k$ is the class of all the classes each of which contains one and but one term of each class in $k$. [Italics present in the original; bold emphasis added.]




Although this paper does not state an explicit definition of "disjoint" for more than two "classes", the construction described in the quote requires "disjoint" to mean pairwise disjoint.



The second earliest use I found is in a paper "Differentiation with Respect to a Function of Limited Variation" (1918). This gives an explicit definition of "disjoint intervals" as "intervals with no points common to any two".



When "pairwise disjoint" appeared, it was not meant to change the meaning of "disjoint", but rather to clarify that the author's meaning of "disjoint" was always pairwise. The earliest use I found is "Integral Forms and Variational Orthogonality" (1938):




"Let $U = U_epsilon = sum_j=1^infty [u_j, u'_j)$ be an enumerable set of pairwise disjoint, half-open intervals which contains $S$."




Here, "pairwise disjoint" has the modern meaning we expect. Yet the same paper also uses "disjoint" by itself to mean the same thing as "pairwise disjoint":




if $S$ is a set consisting of a finite number of disjoint half-open intervals $[u_j, u'_j), j = 1, ..., s$




This usage of "disjoint" occurs earlier in the paper than "pairwise disjoint". To this author, the two terms are not contrasting, but synonymous. This view is the same view that you expressed in your question: "pairwise disjoint" is already the default meaning of "disjoint", but can optionally be added as extra clarification in case the reader believes in the other potential interpretation of "disjoint".






share|cite|improve this answer











$endgroup$








  • 1




    $begingroup$
    Thanks for the great research work. I do not agree with your conclusion about the first citation ("The Thesis of Modern Logistic" by C. J. Keyser). What is being described is defining multiplication of cardinals by taking essentially the cardinal of a Cartesian product of "factors" (sets), but using sets (classes) rather than tuples to form the product. For a set containing exactly one element from each factor to faithfully represent an element of the Cartesian product of the factors, it is essential that no two factors have an element in common. So: the factors must be pairwise disjoint.
    $endgroup$
    – Marc van Leeuwen
    Mar 11 at 11:04











  • $begingroup$
    ... What I just said is not quite exact: if two or more factors share an element, then the problem is not so much that a set containing exactly one element from each factor fails to identify an element of their Cartesian product, the problem is that some elements of the Cartesian product (i.e., some tuples) cannot be represented by such a set. This is because the set of components of the tuple would have more than one element in common with certain factors, so fail the requirement. But it remains: the factors need to be pairwise disjoint for the construction to define cardinal multiplication.
    $endgroup$
    – Marc van Leeuwen
    Mar 11 at 11:14










  • $begingroup$
    For concreteness, $k=0,0,1,1$ is not a multiplicative class, though $bigcap k =emptyset$. Indeed the "class of all the classes each of which contains one and but one term of each class in $k$" is empty, as one easily sees; however the cardinal multiplication should result in a class with $1times2times1=2$ members. For a disjoint class of classes, one would get the right cardinal for the multiplicative class. Final remark: maybe only multiplication of $2$ cardinals is intended here; then of course "disjoint" only applies to two sets and one gets no verdict at all from it
    $endgroup$
    – Marc van Leeuwen
    Mar 11 at 11:36











  • $begingroup$
    @MarcvanLeeuwen I agree - thanks for the important correction. It definitely means pairwise in the first quote. I edited the answer.
    $endgroup$
    – echinodermata
    Mar 15 at 5:48


















7












$begingroup$

In this context disjoint means $A cap B cap C = emptyset$.






share|cite|improve this answer









$endgroup$








  • 12




    $begingroup$
    Is that a standard meaning? I never saw the term formally defined that way in four years of undergrad math classes.
    $endgroup$
    – Connor Harris
    Mar 8 at 15:52






  • 1




    $begingroup$
    me neither, but four people have answered the question this way in four minutes!
    $endgroup$
    – John Lawrence Aspden
    Mar 8 at 15:57






  • 2




    $begingroup$
    If you define "disjoint" to mean "empty intersection" (which is the standard definition) then formally for a family of sets "disjoint" would mean the intersection of the entire family is empty unless stated otherwise. The use of the pleonastic term "pairwise" helps to avoid confusion.
    $endgroup$
    – Umberto P.
    Mar 8 at 16:06







  • 1




    $begingroup$
    This is not the standard definition of disjoint. Google it if you like.
    $endgroup$
    – Marc van Leeuwen
    Mar 8 at 19:47






  • 5




    $begingroup$
    @Brilliand Indeed, somebody trying to find evidence for the fact that disjoint means empty intersection might land on the current page and find plenty of evidence to support it. But if you bluntly proclaim as this answer does that this is what the term means than it does not hurt to check that you can find evidence of a text using it that way, and in spite of the multitude of answers here in the same sense, nobody has been able to do that.
    $endgroup$
    – Marc van Leeuwen
    Mar 9 at 5:54


















4












$begingroup$

Consider the sets $A = 1,2$, $B = 2,3$, $C = 3, 1$. Then $Acap Bcap C = varnothing$, but $A,B,C$ are not pairwise disjoint.






share|cite|improve this answer









$endgroup$




















    0












    $begingroup$

    If disjoint is understood as having an empty intersection, then disjoint sets are not necessarily pairwise disjoint. Pairwise disjoint avoid any doubt, at no cost.






    share|cite|improve this answer









    $endgroup$













      Your Answer





      StackExchange.ifUsing("editor", function ()
      return StackExchange.using("mathjaxEditing", function ()
      StackExchange.MarkdownEditor.creationCallbacks.add(function (editor, postfix)
      StackExchange.mathjaxEditing.prepareWmdForMathJax(editor, postfix, [["$", "$"], ["\\(","\\)"]]);
      );
      );
      , "mathjax-editing");

      StackExchange.ready(function()
      var channelOptions =
      tags: "".split(" "),
      id: "69"
      ;
      initTagRenderer("".split(" "), "".split(" "), channelOptions);

      StackExchange.using("externalEditor", function()
      // Have to fire editor after snippets, if snippets enabled
      if (StackExchange.settings.snippets.snippetsEnabled)
      StackExchange.using("snippets", function()
      createEditor();
      );

      else
      createEditor();

      );

      function createEditor()
      StackExchange.prepareEditor(
      heartbeatType: 'answer',
      autoActivateHeartbeat: false,
      convertImagesToLinks: true,
      noModals: true,
      showLowRepImageUploadWarning: true,
      reputationToPostImages: 10,
      bindNavPrevention: true,
      postfix: "",
      imageUploader:
      brandingHtml: "Powered by u003ca class="icon-imgur-white" href="https://imgur.com/"u003eu003c/au003e",
      contentPolicyHtml: "User contributions licensed under u003ca href="https://creativecommons.org/licenses/by-sa/3.0/"u003ecc by-sa 3.0 with attribution requiredu003c/au003e u003ca href="https://stackoverflow.com/legal/content-policy"u003e(content policy)u003c/au003e",
      allowUrls: true
      ,
      noCode: true, onDemand: true,
      discardSelector: ".discard-answer"
      ,immediatelyShowMarkdownHelp:true
      );



      );













      draft saved

      draft discarded


















      StackExchange.ready(
      function ()
      StackExchange.openid.initPostLogin('.new-post-login', 'https%3a%2f%2fmath.stackexchange.com%2fquestions%2f3140228%2fwhy-do-we-say-pairwise-disjoint-rather-than-disjoint%23new-answer', 'question_page');

      );

      Post as a guest















      Required, but never shown

























      9 Answers
      9






      active

      oldest

      votes








      9 Answers
      9






      active

      oldest

      votes









      active

      oldest

      votes






      active

      oldest

      votes









      21












      $begingroup$

      The word "pairwise" in "pairwise disjoint" is superfluous: a collection of sets is disjoint if no element appears in more than one of the sets at a time, and this means that every pair of distinct sets in the collection has an empty intersection. However including the "pairwise" emphasizes that the property can be checked at the level of pairs from the collection (unlike for instance linear independence of vectors in linear algebra). A "disjoint union" is a union of pairwise disjoint sets; one does not say "pairwise disjoint union".



      To corroborate my point of view, here is a citation from Halmos:




      Pairs of sets with an empty intersection occur frequently enough to justify
      the use of a special word: if $Acap B=emptyset$, the sets $A$ and $B$ are
      called disjoint. The same word is sometimes applied to a collection of sets
      to indicate that any two distinct sets of the collection are disjoint;
      alternatively we may speak in such a situation of a pairwise disjoint
      collection.




      By the way, it is amazing how this site contains lots of answers saying disjoint (for any collection of sets) means empty intersection, like at this question and questions linked from there, and lots of comments saying that is wrong. Which was my motivation to post this as an answer.






      share|cite|improve this answer











      $endgroup$








      • 2




        $begingroup$
        This answer shows another case where the same terminology means something different to different people. To me a disjoint union is a specific formulation of the coproduct in $mathbfSet$ like the definition on Wikpedia rather than a union of pairwise disjoint sets. But this difference is a bit more immaterial because they are isomorphic when the sets involved are pairwise disjoint.
        $endgroup$
        – Potato44
        Mar 9 at 4:48






      • 1




        $begingroup$
        @Potato44: Yes I know, but I did not want to bring up the distinction internal/external disjoint union, which is similar to internal/external direct sum, which has nothing to do with the current question and would be confusing. My point is that nobody says, or should have to say, "pairwise disjoint union".
        $endgroup$
        – Marc van Leeuwen
        Mar 9 at 9:15










      • $begingroup$
        I just checked Royden (4th edition). It uses disjoint to denote what others here call "pairwise disjoint." Same as Karatzas and Shreve, Pugh in his "Real Mathematical Analysis" and Abbott in his "Understanding Analysis." These are the texts I have in a pdf form so I could search quickly. I have never heard or seen anybody calling a collection of sets disjoint to mean empty intersection.
        $endgroup$
        – Mdoc
        Mar 28 at 2:17
















      21












      $begingroup$

      The word "pairwise" in "pairwise disjoint" is superfluous: a collection of sets is disjoint if no element appears in more than one of the sets at a time, and this means that every pair of distinct sets in the collection has an empty intersection. However including the "pairwise" emphasizes that the property can be checked at the level of pairs from the collection (unlike for instance linear independence of vectors in linear algebra). A "disjoint union" is a union of pairwise disjoint sets; one does not say "pairwise disjoint union".



      To corroborate my point of view, here is a citation from Halmos:




      Pairs of sets with an empty intersection occur frequently enough to justify
      the use of a special word: if $Acap B=emptyset$, the sets $A$ and $B$ are
      called disjoint. The same word is sometimes applied to a collection of sets
      to indicate that any two distinct sets of the collection are disjoint;
      alternatively we may speak in such a situation of a pairwise disjoint
      collection.




      By the way, it is amazing how this site contains lots of answers saying disjoint (for any collection of sets) means empty intersection, like at this question and questions linked from there, and lots of comments saying that is wrong. Which was my motivation to post this as an answer.






      share|cite|improve this answer











      $endgroup$








      • 2




        $begingroup$
        This answer shows another case where the same terminology means something different to different people. To me a disjoint union is a specific formulation of the coproduct in $mathbfSet$ like the definition on Wikpedia rather than a union of pairwise disjoint sets. But this difference is a bit more immaterial because they are isomorphic when the sets involved are pairwise disjoint.
        $endgroup$
        – Potato44
        Mar 9 at 4:48






      • 1




        $begingroup$
        @Potato44: Yes I know, but I did not want to bring up the distinction internal/external disjoint union, which is similar to internal/external direct sum, which has nothing to do with the current question and would be confusing. My point is that nobody says, or should have to say, "pairwise disjoint union".
        $endgroup$
        – Marc van Leeuwen
        Mar 9 at 9:15










      • $begingroup$
        I just checked Royden (4th edition). It uses disjoint to denote what others here call "pairwise disjoint." Same as Karatzas and Shreve, Pugh in his "Real Mathematical Analysis" and Abbott in his "Understanding Analysis." These are the texts I have in a pdf form so I could search quickly. I have never heard or seen anybody calling a collection of sets disjoint to mean empty intersection.
        $endgroup$
        – Mdoc
        Mar 28 at 2:17














      21












      21








      21





      $begingroup$

      The word "pairwise" in "pairwise disjoint" is superfluous: a collection of sets is disjoint if no element appears in more than one of the sets at a time, and this means that every pair of distinct sets in the collection has an empty intersection. However including the "pairwise" emphasizes that the property can be checked at the level of pairs from the collection (unlike for instance linear independence of vectors in linear algebra). A "disjoint union" is a union of pairwise disjoint sets; one does not say "pairwise disjoint union".



      To corroborate my point of view, here is a citation from Halmos:




      Pairs of sets with an empty intersection occur frequently enough to justify
      the use of a special word: if $Acap B=emptyset$, the sets $A$ and $B$ are
      called disjoint. The same word is sometimes applied to a collection of sets
      to indicate that any two distinct sets of the collection are disjoint;
      alternatively we may speak in such a situation of a pairwise disjoint
      collection.




      By the way, it is amazing how this site contains lots of answers saying disjoint (for any collection of sets) means empty intersection, like at this question and questions linked from there, and lots of comments saying that is wrong. Which was my motivation to post this as an answer.






      share|cite|improve this answer











      $endgroup$



      The word "pairwise" in "pairwise disjoint" is superfluous: a collection of sets is disjoint if no element appears in more than one of the sets at a time, and this means that every pair of distinct sets in the collection has an empty intersection. However including the "pairwise" emphasizes that the property can be checked at the level of pairs from the collection (unlike for instance linear independence of vectors in linear algebra). A "disjoint union" is a union of pairwise disjoint sets; one does not say "pairwise disjoint union".



      To corroborate my point of view, here is a citation from Halmos:




      Pairs of sets with an empty intersection occur frequently enough to justify
      the use of a special word: if $Acap B=emptyset$, the sets $A$ and $B$ are
      called disjoint. The same word is sometimes applied to a collection of sets
      to indicate that any two distinct sets of the collection are disjoint;
      alternatively we may speak in such a situation of a pairwise disjoint
      collection.




      By the way, it is amazing how this site contains lots of answers saying disjoint (for any collection of sets) means empty intersection, like at this question and questions linked from there, and lots of comments saying that is wrong. Which was my motivation to post this as an answer.







      share|cite|improve this answer














      share|cite|improve this answer



      share|cite|improve this answer








      edited Mar 9 at 12:41

























      answered Mar 8 at 20:03









      Marc van LeeuwenMarc van Leeuwen

      88.7k5111230




      88.7k5111230







      • 2




        $begingroup$
        This answer shows another case where the same terminology means something different to different people. To me a disjoint union is a specific formulation of the coproduct in $mathbfSet$ like the definition on Wikpedia rather than a union of pairwise disjoint sets. But this difference is a bit more immaterial because they are isomorphic when the sets involved are pairwise disjoint.
        $endgroup$
        – Potato44
        Mar 9 at 4:48






      • 1




        $begingroup$
        @Potato44: Yes I know, but I did not want to bring up the distinction internal/external disjoint union, which is similar to internal/external direct sum, which has nothing to do with the current question and would be confusing. My point is that nobody says, or should have to say, "pairwise disjoint union".
        $endgroup$
        – Marc van Leeuwen
        Mar 9 at 9:15










      • $begingroup$
        I just checked Royden (4th edition). It uses disjoint to denote what others here call "pairwise disjoint." Same as Karatzas and Shreve, Pugh in his "Real Mathematical Analysis" and Abbott in his "Understanding Analysis." These are the texts I have in a pdf form so I could search quickly. I have never heard or seen anybody calling a collection of sets disjoint to mean empty intersection.
        $endgroup$
        – Mdoc
        Mar 28 at 2:17













      • 2




        $begingroup$
        This answer shows another case where the same terminology means something different to different people. To me a disjoint union is a specific formulation of the coproduct in $mathbfSet$ like the definition on Wikpedia rather than a union of pairwise disjoint sets. But this difference is a bit more immaterial because they are isomorphic when the sets involved are pairwise disjoint.
        $endgroup$
        – Potato44
        Mar 9 at 4:48






      • 1




        $begingroup$
        @Potato44: Yes I know, but I did not want to bring up the distinction internal/external disjoint union, which is similar to internal/external direct sum, which has nothing to do with the current question and would be confusing. My point is that nobody says, or should have to say, "pairwise disjoint union".
        $endgroup$
        – Marc van Leeuwen
        Mar 9 at 9:15










      • $begingroup$
        I just checked Royden (4th edition). It uses disjoint to denote what others here call "pairwise disjoint." Same as Karatzas and Shreve, Pugh in his "Real Mathematical Analysis" and Abbott in his "Understanding Analysis." These are the texts I have in a pdf form so I could search quickly. I have never heard or seen anybody calling a collection of sets disjoint to mean empty intersection.
        $endgroup$
        – Mdoc
        Mar 28 at 2:17








      2




      2




      $begingroup$
      This answer shows another case where the same terminology means something different to different people. To me a disjoint union is a specific formulation of the coproduct in $mathbfSet$ like the definition on Wikpedia rather than a union of pairwise disjoint sets. But this difference is a bit more immaterial because they are isomorphic when the sets involved are pairwise disjoint.
      $endgroup$
      – Potato44
      Mar 9 at 4:48




      $begingroup$
      This answer shows another case where the same terminology means something different to different people. To me a disjoint union is a specific formulation of the coproduct in $mathbfSet$ like the definition on Wikpedia rather than a union of pairwise disjoint sets. But this difference is a bit more immaterial because they are isomorphic when the sets involved are pairwise disjoint.
      $endgroup$
      – Potato44
      Mar 9 at 4:48




      1




      1




      $begingroup$
      @Potato44: Yes I know, but I did not want to bring up the distinction internal/external disjoint union, which is similar to internal/external direct sum, which has nothing to do with the current question and would be confusing. My point is that nobody says, or should have to say, "pairwise disjoint union".
      $endgroup$
      – Marc van Leeuwen
      Mar 9 at 9:15




      $begingroup$
      @Potato44: Yes I know, but I did not want to bring up the distinction internal/external disjoint union, which is similar to internal/external direct sum, which has nothing to do with the current question and would be confusing. My point is that nobody says, or should have to say, "pairwise disjoint union".
      $endgroup$
      – Marc van Leeuwen
      Mar 9 at 9:15












      $begingroup$
      I just checked Royden (4th edition). It uses disjoint to denote what others here call "pairwise disjoint." Same as Karatzas and Shreve, Pugh in his "Real Mathematical Analysis" and Abbott in his "Understanding Analysis." These are the texts I have in a pdf form so I could search quickly. I have never heard or seen anybody calling a collection of sets disjoint to mean empty intersection.
      $endgroup$
      – Mdoc
      Mar 28 at 2:17





      $begingroup$
      I just checked Royden (4th edition). It uses disjoint to denote what others here call "pairwise disjoint." Same as Karatzas and Shreve, Pugh in his "Real Mathematical Analysis" and Abbott in his "Understanding Analysis." These are the texts I have in a pdf form so I could search quickly. I have never heard or seen anybody calling a collection of sets disjoint to mean empty intersection.
      $endgroup$
      – Mdoc
      Mar 28 at 2:17












      35












      $begingroup$

      As evidenced by the answers and comments on this page, the term "disjoint" is ambiguous - some use it to mean "pairwise disjoint", others use it to mean "empty intersection".



      Thus, for the sake of clarity, I'd recommend avoiding "disjoint" and using "pairwise disjoint".






      share|cite|improve this answer









      $endgroup$








      • 4




        $begingroup$
        Could you back up the claim that 'others use it to mean "empty intersection"' (in the case of more than two sets) with any mathematical text (not with answers on this site, which are indeed east to find)?
        $endgroup$
        – Marc van Leeuwen
        Mar 9 at 12:30






      • 1




        $begingroup$
        I agree with your conclusion, and with @MarcvanLeeuwen's comment: I don't think I've ever seen "disjoint" alone for more than $2$ sets, and I think it would be really weird.
        $endgroup$
        – Arnaud Mortier
        Mar 10 at 0:39















      35












      $begingroup$

      As evidenced by the answers and comments on this page, the term "disjoint" is ambiguous - some use it to mean "pairwise disjoint", others use it to mean "empty intersection".



      Thus, for the sake of clarity, I'd recommend avoiding "disjoint" and using "pairwise disjoint".






      share|cite|improve this answer









      $endgroup$








      • 4




        $begingroup$
        Could you back up the claim that 'others use it to mean "empty intersection"' (in the case of more than two sets) with any mathematical text (not with answers on this site, which are indeed east to find)?
        $endgroup$
        – Marc van Leeuwen
        Mar 9 at 12:30






      • 1




        $begingroup$
        I agree with your conclusion, and with @MarcvanLeeuwen's comment: I don't think I've ever seen "disjoint" alone for more than $2$ sets, and I think it would be really weird.
        $endgroup$
        – Arnaud Mortier
        Mar 10 at 0:39













      35












      35








      35





      $begingroup$

      As evidenced by the answers and comments on this page, the term "disjoint" is ambiguous - some use it to mean "pairwise disjoint", others use it to mean "empty intersection".



      Thus, for the sake of clarity, I'd recommend avoiding "disjoint" and using "pairwise disjoint".






      share|cite|improve this answer









      $endgroup$



      As evidenced by the answers and comments on this page, the term "disjoint" is ambiguous - some use it to mean "pairwise disjoint", others use it to mean "empty intersection".



      Thus, for the sake of clarity, I'd recommend avoiding "disjoint" and using "pairwise disjoint".







      share|cite|improve this answer












      share|cite|improve this answer



      share|cite|improve this answer










      answered Mar 8 at 20:42









      BlueRaja - Danny PflughoeftBlueRaja - Danny Pflughoeft

      5,87342943




      5,87342943







      • 4




        $begingroup$
        Could you back up the claim that 'others use it to mean "empty intersection"' (in the case of more than two sets) with any mathematical text (not with answers on this site, which are indeed east to find)?
        $endgroup$
        – Marc van Leeuwen
        Mar 9 at 12:30






      • 1




        $begingroup$
        I agree with your conclusion, and with @MarcvanLeeuwen's comment: I don't think I've ever seen "disjoint" alone for more than $2$ sets, and I think it would be really weird.
        $endgroup$
        – Arnaud Mortier
        Mar 10 at 0:39












      • 4




        $begingroup$
        Could you back up the claim that 'others use it to mean "empty intersection"' (in the case of more than two sets) with any mathematical text (not with answers on this site, which are indeed east to find)?
        $endgroup$
        – Marc van Leeuwen
        Mar 9 at 12:30






      • 1




        $begingroup$
        I agree with your conclusion, and with @MarcvanLeeuwen's comment: I don't think I've ever seen "disjoint" alone for more than $2$ sets, and I think it would be really weird.
        $endgroup$
        – Arnaud Mortier
        Mar 10 at 0:39







      4




      4




      $begingroup$
      Could you back up the claim that 'others use it to mean "empty intersection"' (in the case of more than two sets) with any mathematical text (not with answers on this site, which are indeed east to find)?
      $endgroup$
      – Marc van Leeuwen
      Mar 9 at 12:30




      $begingroup$
      Could you back up the claim that 'others use it to mean "empty intersection"' (in the case of more than two sets) with any mathematical text (not with answers on this site, which are indeed east to find)?
      $endgroup$
      – Marc van Leeuwen
      Mar 9 at 12:30




      1




      1




      $begingroup$
      I agree with your conclusion, and with @MarcvanLeeuwen's comment: I don't think I've ever seen "disjoint" alone for more than $2$ sets, and I think it would be really weird.
      $endgroup$
      – Arnaud Mortier
      Mar 10 at 0:39




      $begingroup$
      I agree with your conclusion, and with @MarcvanLeeuwen's comment: I don't think I've ever seen "disjoint" alone for more than $2$ sets, and I think it would be really weird.
      $endgroup$
      – Arnaud Mortier
      Mar 10 at 0:39











      33












      $begingroup$

      $1,2,2,3,1,3$ are disjoint but not pairwise disjoint.






      share|cite|improve this answer









      $endgroup$








      • 20




        $begingroup$
        Really? Who would call those disjoint sets?
        $endgroup$
        – John Lawrence Aspden
        Mar 8 at 15:52






      • 38




        $begingroup$
        Everyone. Disjoint means their intersection is empty.
        $endgroup$
        – saulspatz
        Mar 8 at 15:53






      • 16




        $begingroup$
        If sets $A_1,A_2,dots,A_n$ are said to be disjoint then usually it is meant that the sets are pairwise disjoint. See Wolfram for instance. In the other case one says simply that the sets have an empty intersection.
        $endgroup$
        – drhab
        Mar 8 at 16:16







      • 22




        $begingroup$
        So be it. But actually I am the living proof that "everyone" in your first comment is not correct. Lots of times I heard "them" say things like: "we have $mu(bigcup_n=1^inftyA_n)=sum_n=1^inftymu(A_n)$ if the sets are disjoint" (so leaving out pairwise). I do that myself too. It is a good thing however to have a proper definition of this excluding ambiguities. So let's look at this as abuse of language.
        $endgroup$
        – drhab
        Mar 8 at 16:37







      • 17




        $begingroup$
        These sets are not disjoint, at least not by the modern usage of the word “disjoint,” which afaik is used interchangeably with “pairwise disjoint.”
        $endgroup$
        – Shalop
        Mar 8 at 17:33
















      33












      $begingroup$

      $1,2,2,3,1,3$ are disjoint but not pairwise disjoint.






      share|cite|improve this answer









      $endgroup$








      • 20




        $begingroup$
        Really? Who would call those disjoint sets?
        $endgroup$
        – John Lawrence Aspden
        Mar 8 at 15:52






      • 38




        $begingroup$
        Everyone. Disjoint means their intersection is empty.
        $endgroup$
        – saulspatz
        Mar 8 at 15:53






      • 16




        $begingroup$
        If sets $A_1,A_2,dots,A_n$ are said to be disjoint then usually it is meant that the sets are pairwise disjoint. See Wolfram for instance. In the other case one says simply that the sets have an empty intersection.
        $endgroup$
        – drhab
        Mar 8 at 16:16







      • 22




        $begingroup$
        So be it. But actually I am the living proof that "everyone" in your first comment is not correct. Lots of times I heard "them" say things like: "we have $mu(bigcup_n=1^inftyA_n)=sum_n=1^inftymu(A_n)$ if the sets are disjoint" (so leaving out pairwise). I do that myself too. It is a good thing however to have a proper definition of this excluding ambiguities. So let's look at this as abuse of language.
        $endgroup$
        – drhab
        Mar 8 at 16:37







      • 17




        $begingroup$
        These sets are not disjoint, at least not by the modern usage of the word “disjoint,” which afaik is used interchangeably with “pairwise disjoint.”
        $endgroup$
        – Shalop
        Mar 8 at 17:33














      33












      33








      33





      $begingroup$

      $1,2,2,3,1,3$ are disjoint but not pairwise disjoint.






      share|cite|improve this answer









      $endgroup$



      $1,2,2,3,1,3$ are disjoint but not pairwise disjoint.







      share|cite|improve this answer












      share|cite|improve this answer



      share|cite|improve this answer










      answered Mar 8 at 15:51









      saulspatzsaulspatz

      17.2k31435




      17.2k31435







      • 20




        $begingroup$
        Really? Who would call those disjoint sets?
        $endgroup$
        – John Lawrence Aspden
        Mar 8 at 15:52






      • 38




        $begingroup$
        Everyone. Disjoint means their intersection is empty.
        $endgroup$
        – saulspatz
        Mar 8 at 15:53






      • 16




        $begingroup$
        If sets $A_1,A_2,dots,A_n$ are said to be disjoint then usually it is meant that the sets are pairwise disjoint. See Wolfram for instance. In the other case one says simply that the sets have an empty intersection.
        $endgroup$
        – drhab
        Mar 8 at 16:16







      • 22




        $begingroup$
        So be it. But actually I am the living proof that "everyone" in your first comment is not correct. Lots of times I heard "them" say things like: "we have $mu(bigcup_n=1^inftyA_n)=sum_n=1^inftymu(A_n)$ if the sets are disjoint" (so leaving out pairwise). I do that myself too. It is a good thing however to have a proper definition of this excluding ambiguities. So let's look at this as abuse of language.
        $endgroup$
        – drhab
        Mar 8 at 16:37







      • 17




        $begingroup$
        These sets are not disjoint, at least not by the modern usage of the word “disjoint,” which afaik is used interchangeably with “pairwise disjoint.”
        $endgroup$
        – Shalop
        Mar 8 at 17:33













      • 20




        $begingroup$
        Really? Who would call those disjoint sets?
        $endgroup$
        – John Lawrence Aspden
        Mar 8 at 15:52






      • 38




        $begingroup$
        Everyone. Disjoint means their intersection is empty.
        $endgroup$
        – saulspatz
        Mar 8 at 15:53






      • 16




        $begingroup$
        If sets $A_1,A_2,dots,A_n$ are said to be disjoint then usually it is meant that the sets are pairwise disjoint. See Wolfram for instance. In the other case one says simply that the sets have an empty intersection.
        $endgroup$
        – drhab
        Mar 8 at 16:16







      • 22




        $begingroup$
        So be it. But actually I am the living proof that "everyone" in your first comment is not correct. Lots of times I heard "them" say things like: "we have $mu(bigcup_n=1^inftyA_n)=sum_n=1^inftymu(A_n)$ if the sets are disjoint" (so leaving out pairwise). I do that myself too. It is a good thing however to have a proper definition of this excluding ambiguities. So let's look at this as abuse of language.
        $endgroup$
        – drhab
        Mar 8 at 16:37







      • 17




        $begingroup$
        These sets are not disjoint, at least not by the modern usage of the word “disjoint,” which afaik is used interchangeably with “pairwise disjoint.”
        $endgroup$
        – Shalop
        Mar 8 at 17:33








      20




      20




      $begingroup$
      Really? Who would call those disjoint sets?
      $endgroup$
      – John Lawrence Aspden
      Mar 8 at 15:52




      $begingroup$
      Really? Who would call those disjoint sets?
      $endgroup$
      – John Lawrence Aspden
      Mar 8 at 15:52




      38




      38




      $begingroup$
      Everyone. Disjoint means their intersection is empty.
      $endgroup$
      – saulspatz
      Mar 8 at 15:53




      $begingroup$
      Everyone. Disjoint means their intersection is empty.
      $endgroup$
      – saulspatz
      Mar 8 at 15:53




      16




      16




      $begingroup$
      If sets $A_1,A_2,dots,A_n$ are said to be disjoint then usually it is meant that the sets are pairwise disjoint. See Wolfram for instance. In the other case one says simply that the sets have an empty intersection.
      $endgroup$
      – drhab
      Mar 8 at 16:16





      $begingroup$
      If sets $A_1,A_2,dots,A_n$ are said to be disjoint then usually it is meant that the sets are pairwise disjoint. See Wolfram for instance. In the other case one says simply that the sets have an empty intersection.
      $endgroup$
      – drhab
      Mar 8 at 16:16





      22




      22




      $begingroup$
      So be it. But actually I am the living proof that "everyone" in your first comment is not correct. Lots of times I heard "them" say things like: "we have $mu(bigcup_n=1^inftyA_n)=sum_n=1^inftymu(A_n)$ if the sets are disjoint" (so leaving out pairwise). I do that myself too. It is a good thing however to have a proper definition of this excluding ambiguities. So let's look at this as abuse of language.
      $endgroup$
      – drhab
      Mar 8 at 16:37





      $begingroup$
      So be it. But actually I am the living proof that "everyone" in your first comment is not correct. Lots of times I heard "them" say things like: "we have $mu(bigcup_n=1^inftyA_n)=sum_n=1^inftymu(A_n)$ if the sets are disjoint" (so leaving out pairwise). I do that myself too. It is a good thing however to have a proper definition of this excluding ambiguities. So let's look at this as abuse of language.
      $endgroup$
      – drhab
      Mar 8 at 16:37





      17




      17




      $begingroup$
      These sets are not disjoint, at least not by the modern usage of the word “disjoint,” which afaik is used interchangeably with “pairwise disjoint.”
      $endgroup$
      – Shalop
      Mar 8 at 17:33





      $begingroup$
      These sets are not disjoint, at least not by the modern usage of the word “disjoint,” which afaik is used interchangeably with “pairwise disjoint.”
      $endgroup$
      – Shalop
      Mar 8 at 17:33












      15












      $begingroup$

      Two sets are disjoint when their intersection is empty. Sets are pairwise disjoint when any two of them are disjoint. Most if not all mathematicians also call such sets disjoint, making pairwise a superfluous term for emphasis.






      share|cite|improve this answer











      $endgroup$








      • 6




        $begingroup$
        I have never seen this sense of disjoint for more than two sets been defined or used in mathematics. The Wikipedia article for disjoint sets does not mention it, and the notion of "disjoint union" definitely refers to pairwise disjoint sets.
        $endgroup$
        – Marc van Leeuwen
        Mar 8 at 19:45






      • 1




        $begingroup$
        @MarcvanLeeuwen Three-or-more sets that are disjoint-but-not-pairwise-disjoint in the sense I describe probably warrant some name, but as you note it's not usually the nomenclature I mentioned, probably because it makes "pairwise" compulsory more than anyone wants to write it.
        $endgroup$
        – J.G.
        Mar 8 at 19:51







      • 4




        $begingroup$
        @MarcvanLeeuwen From the second sentence in the Wikipedia article: "disjoint sets are sets whose intersection is the empty set." If this definition was solely meant for pairs of sets, I would expect it to say "two sets are disjoint if..." rather than "disjoint sets are...".
        $endgroup$
        – Ryan
        Mar 9 at 4:49






      • 1




        $begingroup$
        @Ryan: Thank you for that comment, it allowed me to find the citation of Halmos that I added to my answer (the one provided in the footnote to your WP quote) and which clearly shows that the WP quote is incorrectly phrased. Elsewhere in the WP article (like in the opening phrase) they are careful to always specify intersection of two sets. I'll correct the WP intro.
        $endgroup$
        – Marc van Leeuwen
        Mar 9 at 7:50






      • 1




        $begingroup$
        @MarcvanLeeuwen I've amended my answer to reflect the usage consensus.
        $endgroup$
        – J.G.
        Mar 9 at 8:17















      15












      $begingroup$

      Two sets are disjoint when their intersection is empty. Sets are pairwise disjoint when any two of them are disjoint. Most if not all mathematicians also call such sets disjoint, making pairwise a superfluous term for emphasis.






      share|cite|improve this answer











      $endgroup$








      • 6




        $begingroup$
        I have never seen this sense of disjoint for more than two sets been defined or used in mathematics. The Wikipedia article for disjoint sets does not mention it, and the notion of "disjoint union" definitely refers to pairwise disjoint sets.
        $endgroup$
        – Marc van Leeuwen
        Mar 8 at 19:45






      • 1




        $begingroup$
        @MarcvanLeeuwen Three-or-more sets that are disjoint-but-not-pairwise-disjoint in the sense I describe probably warrant some name, but as you note it's not usually the nomenclature I mentioned, probably because it makes "pairwise" compulsory more than anyone wants to write it.
        $endgroup$
        – J.G.
        Mar 8 at 19:51







      • 4




        $begingroup$
        @MarcvanLeeuwen From the second sentence in the Wikipedia article: "disjoint sets are sets whose intersection is the empty set." If this definition was solely meant for pairs of sets, I would expect it to say "two sets are disjoint if..." rather than "disjoint sets are...".
        $endgroup$
        – Ryan
        Mar 9 at 4:49






      • 1




        $begingroup$
        @Ryan: Thank you for that comment, it allowed me to find the citation of Halmos that I added to my answer (the one provided in the footnote to your WP quote) and which clearly shows that the WP quote is incorrectly phrased. Elsewhere in the WP article (like in the opening phrase) they are careful to always specify intersection of two sets. I'll correct the WP intro.
        $endgroup$
        – Marc van Leeuwen
        Mar 9 at 7:50






      • 1




        $begingroup$
        @MarcvanLeeuwen I've amended my answer to reflect the usage consensus.
        $endgroup$
        – J.G.
        Mar 9 at 8:17













      15












      15








      15





      $begingroup$

      Two sets are disjoint when their intersection is empty. Sets are pairwise disjoint when any two of them are disjoint. Most if not all mathematicians also call such sets disjoint, making pairwise a superfluous term for emphasis.






      share|cite|improve this answer











      $endgroup$



      Two sets are disjoint when their intersection is empty. Sets are pairwise disjoint when any two of them are disjoint. Most if not all mathematicians also call such sets disjoint, making pairwise a superfluous term for emphasis.







      share|cite|improve this answer














      share|cite|improve this answer



      share|cite|improve this answer








      edited Mar 9 at 8:16

























      answered Mar 8 at 15:53









      J.G.J.G.

      32.6k23250




      32.6k23250







      • 6




        $begingroup$
        I have never seen this sense of disjoint for more than two sets been defined or used in mathematics. The Wikipedia article for disjoint sets does not mention it, and the notion of "disjoint union" definitely refers to pairwise disjoint sets.
        $endgroup$
        – Marc van Leeuwen
        Mar 8 at 19:45






      • 1




        $begingroup$
        @MarcvanLeeuwen Three-or-more sets that are disjoint-but-not-pairwise-disjoint in the sense I describe probably warrant some name, but as you note it's not usually the nomenclature I mentioned, probably because it makes "pairwise" compulsory more than anyone wants to write it.
        $endgroup$
        – J.G.
        Mar 8 at 19:51







      • 4




        $begingroup$
        @MarcvanLeeuwen From the second sentence in the Wikipedia article: "disjoint sets are sets whose intersection is the empty set." If this definition was solely meant for pairs of sets, I would expect it to say "two sets are disjoint if..." rather than "disjoint sets are...".
        $endgroup$
        – Ryan
        Mar 9 at 4:49






      • 1




        $begingroup$
        @Ryan: Thank you for that comment, it allowed me to find the citation of Halmos that I added to my answer (the one provided in the footnote to your WP quote) and which clearly shows that the WP quote is incorrectly phrased. Elsewhere in the WP article (like in the opening phrase) they are careful to always specify intersection of two sets. I'll correct the WP intro.
        $endgroup$
        – Marc van Leeuwen
        Mar 9 at 7:50






      • 1




        $begingroup$
        @MarcvanLeeuwen I've amended my answer to reflect the usage consensus.
        $endgroup$
        – J.G.
        Mar 9 at 8:17












      • 6




        $begingroup$
        I have never seen this sense of disjoint for more than two sets been defined or used in mathematics. The Wikipedia article for disjoint sets does not mention it, and the notion of "disjoint union" definitely refers to pairwise disjoint sets.
        $endgroup$
        – Marc van Leeuwen
        Mar 8 at 19:45






      • 1




        $begingroup$
        @MarcvanLeeuwen Three-or-more sets that are disjoint-but-not-pairwise-disjoint in the sense I describe probably warrant some name, but as you note it's not usually the nomenclature I mentioned, probably because it makes "pairwise" compulsory more than anyone wants to write it.
        $endgroup$
        – J.G.
        Mar 8 at 19:51







      • 4




        $begingroup$
        @MarcvanLeeuwen From the second sentence in the Wikipedia article: "disjoint sets are sets whose intersection is the empty set." If this definition was solely meant for pairs of sets, I would expect it to say "two sets are disjoint if..." rather than "disjoint sets are...".
        $endgroup$
        – Ryan
        Mar 9 at 4:49






      • 1




        $begingroup$
        @Ryan: Thank you for that comment, it allowed me to find the citation of Halmos that I added to my answer (the one provided in the footnote to your WP quote) and which clearly shows that the WP quote is incorrectly phrased. Elsewhere in the WP article (like in the opening phrase) they are careful to always specify intersection of two sets. I'll correct the WP intro.
        $endgroup$
        – Marc van Leeuwen
        Mar 9 at 7:50






      • 1




        $begingroup$
        @MarcvanLeeuwen I've amended my answer to reflect the usage consensus.
        $endgroup$
        – J.G.
        Mar 9 at 8:17







      6




      6




      $begingroup$
      I have never seen this sense of disjoint for more than two sets been defined or used in mathematics. The Wikipedia article for disjoint sets does not mention it, and the notion of "disjoint union" definitely refers to pairwise disjoint sets.
      $endgroup$
      – Marc van Leeuwen
      Mar 8 at 19:45




      $begingroup$
      I have never seen this sense of disjoint for more than two sets been defined or used in mathematics. The Wikipedia article for disjoint sets does not mention it, and the notion of "disjoint union" definitely refers to pairwise disjoint sets.
      $endgroup$
      – Marc van Leeuwen
      Mar 8 at 19:45




      1




      1




      $begingroup$
      @MarcvanLeeuwen Three-or-more sets that are disjoint-but-not-pairwise-disjoint in the sense I describe probably warrant some name, but as you note it's not usually the nomenclature I mentioned, probably because it makes "pairwise" compulsory more than anyone wants to write it.
      $endgroup$
      – J.G.
      Mar 8 at 19:51





      $begingroup$
      @MarcvanLeeuwen Three-or-more sets that are disjoint-but-not-pairwise-disjoint in the sense I describe probably warrant some name, but as you note it's not usually the nomenclature I mentioned, probably because it makes "pairwise" compulsory more than anyone wants to write it.
      $endgroup$
      – J.G.
      Mar 8 at 19:51





      4




      4




      $begingroup$
      @MarcvanLeeuwen From the second sentence in the Wikipedia article: "disjoint sets are sets whose intersection is the empty set." If this definition was solely meant for pairs of sets, I would expect it to say "two sets are disjoint if..." rather than "disjoint sets are...".
      $endgroup$
      – Ryan
      Mar 9 at 4:49




      $begingroup$
      @MarcvanLeeuwen From the second sentence in the Wikipedia article: "disjoint sets are sets whose intersection is the empty set." If this definition was solely meant for pairs of sets, I would expect it to say "two sets are disjoint if..." rather than "disjoint sets are...".
      $endgroup$
      – Ryan
      Mar 9 at 4:49




      1




      1




      $begingroup$
      @Ryan: Thank you for that comment, it allowed me to find the citation of Halmos that I added to my answer (the one provided in the footnote to your WP quote) and which clearly shows that the WP quote is incorrectly phrased. Elsewhere in the WP article (like in the opening phrase) they are careful to always specify intersection of two sets. I'll correct the WP intro.
      $endgroup$
      – Marc van Leeuwen
      Mar 9 at 7:50




      $begingroup$
      @Ryan: Thank you for that comment, it allowed me to find the citation of Halmos that I added to my answer (the one provided in the footnote to your WP quote) and which clearly shows that the WP quote is incorrectly phrased. Elsewhere in the WP article (like in the opening phrase) they are careful to always specify intersection of two sets. I'll correct the WP intro.
      $endgroup$
      – Marc van Leeuwen
      Mar 9 at 7:50




      1




      1




      $begingroup$
      @MarcvanLeeuwen I've amended my answer to reflect the usage consensus.
      $endgroup$
      – J.G.
      Mar 9 at 8:17




      $begingroup$
      @MarcvanLeeuwen I've amended my answer to reflect the usage consensus.
      $endgroup$
      – J.G.
      Mar 9 at 8:17











      8












      $begingroup$

      Let $A=1,2, B=2,3,C=3,4$. Then the sets are disjoint because $Acap Bcap C=emptyset$, but not pairwise disjoint because you have pairs such as $A,B$ such that $Acap Bnot =emptyset$.






      share|cite|improve this answer











      $endgroup$












      • $begingroup$
        Rats! What is the notation for an empty set?
        $endgroup$
        – Oscar Lanzi
        Mar 8 at 15:54










      • $begingroup$
        Thank you, @jg.
        $endgroup$
        – Oscar Lanzi
        Mar 8 at 15:58






      • 1




        $begingroup$
        I use varnothing, "$varnothing$".
        $endgroup$
        – Eric Towers
        Mar 9 at 22:14















      8












      $begingroup$

      Let $A=1,2, B=2,3,C=3,4$. Then the sets are disjoint because $Acap Bcap C=emptyset$, but not pairwise disjoint because you have pairs such as $A,B$ such that $Acap Bnot =emptyset$.






      share|cite|improve this answer











      $endgroup$












      • $begingroup$
        Rats! What is the notation for an empty set?
        $endgroup$
        – Oscar Lanzi
        Mar 8 at 15:54










      • $begingroup$
        Thank you, @jg.
        $endgroup$
        – Oscar Lanzi
        Mar 8 at 15:58






      • 1




        $begingroup$
        I use varnothing, "$varnothing$".
        $endgroup$
        – Eric Towers
        Mar 9 at 22:14













      8












      8








      8





      $begingroup$

      Let $A=1,2, B=2,3,C=3,4$. Then the sets are disjoint because $Acap Bcap C=emptyset$, but not pairwise disjoint because you have pairs such as $A,B$ such that $Acap Bnot =emptyset$.






      share|cite|improve this answer











      $endgroup$



      Let $A=1,2, B=2,3,C=3,4$. Then the sets are disjoint because $Acap Bcap C=emptyset$, but not pairwise disjoint because you have pairs such as $A,B$ such that $Acap Bnot =emptyset$.







      share|cite|improve this answer














      share|cite|improve this answer



      share|cite|improve this answer








      edited Mar 8 at 15:54









      J.G.

      32.6k23250




      32.6k23250










      answered Mar 8 at 15:53









      Oscar LanziOscar Lanzi

      13.5k12136




      13.5k12136











      • $begingroup$
        Rats! What is the notation for an empty set?
        $endgroup$
        – Oscar Lanzi
        Mar 8 at 15:54










      • $begingroup$
        Thank you, @jg.
        $endgroup$
        – Oscar Lanzi
        Mar 8 at 15:58






      • 1




        $begingroup$
        I use varnothing, "$varnothing$".
        $endgroup$
        – Eric Towers
        Mar 9 at 22:14
















      • $begingroup$
        Rats! What is the notation for an empty set?
        $endgroup$
        – Oscar Lanzi
        Mar 8 at 15:54










      • $begingroup$
        Thank you, @jg.
        $endgroup$
        – Oscar Lanzi
        Mar 8 at 15:58






      • 1




        $begingroup$
        I use varnothing, "$varnothing$".
        $endgroup$
        – Eric Towers
        Mar 9 at 22:14















      $begingroup$
      Rats! What is the notation for an empty set?
      $endgroup$
      – Oscar Lanzi
      Mar 8 at 15:54




      $begingroup$
      Rats! What is the notation for an empty set?
      $endgroup$
      – Oscar Lanzi
      Mar 8 at 15:54












      $begingroup$
      Thank you, @jg.
      $endgroup$
      – Oscar Lanzi
      Mar 8 at 15:58




      $begingroup$
      Thank you, @jg.
      $endgroup$
      – Oscar Lanzi
      Mar 8 at 15:58




      1




      1




      $begingroup$
      I use varnothing, "$varnothing$".
      $endgroup$
      – Eric Towers
      Mar 9 at 22:14




      $begingroup$
      I use varnothing, "$varnothing$".
      $endgroup$
      – Eric Towers
      Mar 9 at 22:14











      8












      $begingroup$

      This is a question about terminology and its usage in practice, so the basis for an answer should come from real quotations and the historical record rather than unsupported opinion. Looking at early known uses of the term, the few sources that I examined all used "disjoint" in its pairwise sense rather than in the sense of having empty intersection.



      The earliest published use (known to me) of the term "disjoint" for its mathematical meaning is in a paper "The Thesis of Modern Logistic" (1909), which is the earliest such use found in a JSTOR search and the earliest such use listed in Earliest Known Uses of
      Some of the Words of Mathematics
      . (Note: Here and throughout, I do not claim that the quoted sources are truly the earliest sources, but merely the earliest ones I was able to find. It is clear that readily available search tools today cover only a small part of historical record. Especially missing from my research is German papers and books, which surely are a large part of the early history of set theory.)



      This first paper uses "disjoint" in its description of a construction similar to what we now call an $n$-ary Cartesian product, but where the members of the product are sets rather than ordered $n$-tuples:




      Multiplication of cardinals is also defined in purely logical terms. This is done by means of the concept (due to Whitehead) of multiplicative class, which is itself given in terms of logical constants: $k$ being a class of disjoint classes, the multiplicative class of $k$ is the class of all the classes each of which contains one and but one term of each class in $k$. [Italics present in the original; bold emphasis added.]




      Although this paper does not state an explicit definition of "disjoint" for more than two "classes", the construction described in the quote requires "disjoint" to mean pairwise disjoint.



      The second earliest use I found is in a paper "Differentiation with Respect to a Function of Limited Variation" (1918). This gives an explicit definition of "disjoint intervals" as "intervals with no points common to any two".



      When "pairwise disjoint" appeared, it was not meant to change the meaning of "disjoint", but rather to clarify that the author's meaning of "disjoint" was always pairwise. The earliest use I found is "Integral Forms and Variational Orthogonality" (1938):




      "Let $U = U_epsilon = sum_j=1^infty [u_j, u'_j)$ be an enumerable set of pairwise disjoint, half-open intervals which contains $S$."




      Here, "pairwise disjoint" has the modern meaning we expect. Yet the same paper also uses "disjoint" by itself to mean the same thing as "pairwise disjoint":




      if $S$ is a set consisting of a finite number of disjoint half-open intervals $[u_j, u'_j), j = 1, ..., s$




      This usage of "disjoint" occurs earlier in the paper than "pairwise disjoint". To this author, the two terms are not contrasting, but synonymous. This view is the same view that you expressed in your question: "pairwise disjoint" is already the default meaning of "disjoint", but can optionally be added as extra clarification in case the reader believes in the other potential interpretation of "disjoint".






      share|cite|improve this answer











      $endgroup$








      • 1




        $begingroup$
        Thanks for the great research work. I do not agree with your conclusion about the first citation ("The Thesis of Modern Logistic" by C. J. Keyser). What is being described is defining multiplication of cardinals by taking essentially the cardinal of a Cartesian product of "factors" (sets), but using sets (classes) rather than tuples to form the product. For a set containing exactly one element from each factor to faithfully represent an element of the Cartesian product of the factors, it is essential that no two factors have an element in common. So: the factors must be pairwise disjoint.
        $endgroup$
        – Marc van Leeuwen
        Mar 11 at 11:04











      • $begingroup$
        ... What I just said is not quite exact: if two or more factors share an element, then the problem is not so much that a set containing exactly one element from each factor fails to identify an element of their Cartesian product, the problem is that some elements of the Cartesian product (i.e., some tuples) cannot be represented by such a set. This is because the set of components of the tuple would have more than one element in common with certain factors, so fail the requirement. But it remains: the factors need to be pairwise disjoint for the construction to define cardinal multiplication.
        $endgroup$
        – Marc van Leeuwen
        Mar 11 at 11:14










      • $begingroup$
        For concreteness, $k=0,0,1,1$ is not a multiplicative class, though $bigcap k =emptyset$. Indeed the "class of all the classes each of which contains one and but one term of each class in $k$" is empty, as one easily sees; however the cardinal multiplication should result in a class with $1times2times1=2$ members. For a disjoint class of classes, one would get the right cardinal for the multiplicative class. Final remark: maybe only multiplication of $2$ cardinals is intended here; then of course "disjoint" only applies to two sets and one gets no verdict at all from it
        $endgroup$
        – Marc van Leeuwen
        Mar 11 at 11:36











      • $begingroup$
        @MarcvanLeeuwen I agree - thanks for the important correction. It definitely means pairwise in the first quote. I edited the answer.
        $endgroup$
        – echinodermata
        Mar 15 at 5:48















      8












      $begingroup$

      This is a question about terminology and its usage in practice, so the basis for an answer should come from real quotations and the historical record rather than unsupported opinion. Looking at early known uses of the term, the few sources that I examined all used "disjoint" in its pairwise sense rather than in the sense of having empty intersection.



      The earliest published use (known to me) of the term "disjoint" for its mathematical meaning is in a paper "The Thesis of Modern Logistic" (1909), which is the earliest such use found in a JSTOR search and the earliest such use listed in Earliest Known Uses of
      Some of the Words of Mathematics
      . (Note: Here and throughout, I do not claim that the quoted sources are truly the earliest sources, but merely the earliest ones I was able to find. It is clear that readily available search tools today cover only a small part of historical record. Especially missing from my research is German papers and books, which surely are a large part of the early history of set theory.)



      This first paper uses "disjoint" in its description of a construction similar to what we now call an $n$-ary Cartesian product, but where the members of the product are sets rather than ordered $n$-tuples:




      Multiplication of cardinals is also defined in purely logical terms. This is done by means of the concept (due to Whitehead) of multiplicative class, which is itself given in terms of logical constants: $k$ being a class of disjoint classes, the multiplicative class of $k$ is the class of all the classes each of which contains one and but one term of each class in $k$. [Italics present in the original; bold emphasis added.]




      Although this paper does not state an explicit definition of "disjoint" for more than two "classes", the construction described in the quote requires "disjoint" to mean pairwise disjoint.



      The second earliest use I found is in a paper "Differentiation with Respect to a Function of Limited Variation" (1918). This gives an explicit definition of "disjoint intervals" as "intervals with no points common to any two".



      When "pairwise disjoint" appeared, it was not meant to change the meaning of "disjoint", but rather to clarify that the author's meaning of "disjoint" was always pairwise. The earliest use I found is "Integral Forms and Variational Orthogonality" (1938):




      "Let $U = U_epsilon = sum_j=1^infty [u_j, u'_j)$ be an enumerable set of pairwise disjoint, half-open intervals which contains $S$."




      Here, "pairwise disjoint" has the modern meaning we expect. Yet the same paper also uses "disjoint" by itself to mean the same thing as "pairwise disjoint":




      if $S$ is a set consisting of a finite number of disjoint half-open intervals $[u_j, u'_j), j = 1, ..., s$




      This usage of "disjoint" occurs earlier in the paper than "pairwise disjoint". To this author, the two terms are not contrasting, but synonymous. This view is the same view that you expressed in your question: "pairwise disjoint" is already the default meaning of "disjoint", but can optionally be added as extra clarification in case the reader believes in the other potential interpretation of "disjoint".






      share|cite|improve this answer











      $endgroup$








      • 1




        $begingroup$
        Thanks for the great research work. I do not agree with your conclusion about the first citation ("The Thesis of Modern Logistic" by C. J. Keyser). What is being described is defining multiplication of cardinals by taking essentially the cardinal of a Cartesian product of "factors" (sets), but using sets (classes) rather than tuples to form the product. For a set containing exactly one element from each factor to faithfully represent an element of the Cartesian product of the factors, it is essential that no two factors have an element in common. So: the factors must be pairwise disjoint.
        $endgroup$
        – Marc van Leeuwen
        Mar 11 at 11:04











      • $begingroup$
        ... What I just said is not quite exact: if two or more factors share an element, then the problem is not so much that a set containing exactly one element from each factor fails to identify an element of their Cartesian product, the problem is that some elements of the Cartesian product (i.e., some tuples) cannot be represented by such a set. This is because the set of components of the tuple would have more than one element in common with certain factors, so fail the requirement. But it remains: the factors need to be pairwise disjoint for the construction to define cardinal multiplication.
        $endgroup$
        – Marc van Leeuwen
        Mar 11 at 11:14










      • $begingroup$
        For concreteness, $k=0,0,1,1$ is not a multiplicative class, though $bigcap k =emptyset$. Indeed the "class of all the classes each of which contains one and but one term of each class in $k$" is empty, as one easily sees; however the cardinal multiplication should result in a class with $1times2times1=2$ members. For a disjoint class of classes, one would get the right cardinal for the multiplicative class. Final remark: maybe only multiplication of $2$ cardinals is intended here; then of course "disjoint" only applies to two sets and one gets no verdict at all from it
        $endgroup$
        – Marc van Leeuwen
        Mar 11 at 11:36











      • $begingroup$
        @MarcvanLeeuwen I agree - thanks for the important correction. It definitely means pairwise in the first quote. I edited the answer.
        $endgroup$
        – echinodermata
        Mar 15 at 5:48













      8












      8








      8





      $begingroup$

      This is a question about terminology and its usage in practice, so the basis for an answer should come from real quotations and the historical record rather than unsupported opinion. Looking at early known uses of the term, the few sources that I examined all used "disjoint" in its pairwise sense rather than in the sense of having empty intersection.



      The earliest published use (known to me) of the term "disjoint" for its mathematical meaning is in a paper "The Thesis of Modern Logistic" (1909), which is the earliest such use found in a JSTOR search and the earliest such use listed in Earliest Known Uses of
      Some of the Words of Mathematics
      . (Note: Here and throughout, I do not claim that the quoted sources are truly the earliest sources, but merely the earliest ones I was able to find. It is clear that readily available search tools today cover only a small part of historical record. Especially missing from my research is German papers and books, which surely are a large part of the early history of set theory.)



      This first paper uses "disjoint" in its description of a construction similar to what we now call an $n$-ary Cartesian product, but where the members of the product are sets rather than ordered $n$-tuples:




      Multiplication of cardinals is also defined in purely logical terms. This is done by means of the concept (due to Whitehead) of multiplicative class, which is itself given in terms of logical constants: $k$ being a class of disjoint classes, the multiplicative class of $k$ is the class of all the classes each of which contains one and but one term of each class in $k$. [Italics present in the original; bold emphasis added.]




      Although this paper does not state an explicit definition of "disjoint" for more than two "classes", the construction described in the quote requires "disjoint" to mean pairwise disjoint.



      The second earliest use I found is in a paper "Differentiation with Respect to a Function of Limited Variation" (1918). This gives an explicit definition of "disjoint intervals" as "intervals with no points common to any two".



      When "pairwise disjoint" appeared, it was not meant to change the meaning of "disjoint", but rather to clarify that the author's meaning of "disjoint" was always pairwise. The earliest use I found is "Integral Forms and Variational Orthogonality" (1938):




      "Let $U = U_epsilon = sum_j=1^infty [u_j, u'_j)$ be an enumerable set of pairwise disjoint, half-open intervals which contains $S$."




      Here, "pairwise disjoint" has the modern meaning we expect. Yet the same paper also uses "disjoint" by itself to mean the same thing as "pairwise disjoint":




      if $S$ is a set consisting of a finite number of disjoint half-open intervals $[u_j, u'_j), j = 1, ..., s$




      This usage of "disjoint" occurs earlier in the paper than "pairwise disjoint". To this author, the two terms are not contrasting, but synonymous. This view is the same view that you expressed in your question: "pairwise disjoint" is already the default meaning of "disjoint", but can optionally be added as extra clarification in case the reader believes in the other potential interpretation of "disjoint".






      share|cite|improve this answer











      $endgroup$



      This is a question about terminology and its usage in practice, so the basis for an answer should come from real quotations and the historical record rather than unsupported opinion. Looking at early known uses of the term, the few sources that I examined all used "disjoint" in its pairwise sense rather than in the sense of having empty intersection.



      The earliest published use (known to me) of the term "disjoint" for its mathematical meaning is in a paper "The Thesis of Modern Logistic" (1909), which is the earliest such use found in a JSTOR search and the earliest such use listed in Earliest Known Uses of
      Some of the Words of Mathematics
      . (Note: Here and throughout, I do not claim that the quoted sources are truly the earliest sources, but merely the earliest ones I was able to find. It is clear that readily available search tools today cover only a small part of historical record. Especially missing from my research is German papers and books, which surely are a large part of the early history of set theory.)



      This first paper uses "disjoint" in its description of a construction similar to what we now call an $n$-ary Cartesian product, but where the members of the product are sets rather than ordered $n$-tuples:




      Multiplication of cardinals is also defined in purely logical terms. This is done by means of the concept (due to Whitehead) of multiplicative class, which is itself given in terms of logical constants: $k$ being a class of disjoint classes, the multiplicative class of $k$ is the class of all the classes each of which contains one and but one term of each class in $k$. [Italics present in the original; bold emphasis added.]




      Although this paper does not state an explicit definition of "disjoint" for more than two "classes", the construction described in the quote requires "disjoint" to mean pairwise disjoint.



      The second earliest use I found is in a paper "Differentiation with Respect to a Function of Limited Variation" (1918). This gives an explicit definition of "disjoint intervals" as "intervals with no points common to any two".



      When "pairwise disjoint" appeared, it was not meant to change the meaning of "disjoint", but rather to clarify that the author's meaning of "disjoint" was always pairwise. The earliest use I found is "Integral Forms and Variational Orthogonality" (1938):




      "Let $U = U_epsilon = sum_j=1^infty [u_j, u'_j)$ be an enumerable set of pairwise disjoint, half-open intervals which contains $S$."




      Here, "pairwise disjoint" has the modern meaning we expect. Yet the same paper also uses "disjoint" by itself to mean the same thing as "pairwise disjoint":




      if $S$ is a set consisting of a finite number of disjoint half-open intervals $[u_j, u'_j), j = 1, ..., s$




      This usage of "disjoint" occurs earlier in the paper than "pairwise disjoint". To this author, the two terms are not contrasting, but synonymous. This view is the same view that you expressed in your question: "pairwise disjoint" is already the default meaning of "disjoint", but can optionally be added as extra clarification in case the reader believes in the other potential interpretation of "disjoint".







      share|cite|improve this answer














      share|cite|improve this answer



      share|cite|improve this answer








      edited Mar 15 at 5:20

























      answered Mar 10 at 20:51









      echinodermataechinodermata

      2,52911228




      2,52911228







      • 1




        $begingroup$
        Thanks for the great research work. I do not agree with your conclusion about the first citation ("The Thesis of Modern Logistic" by C. J. Keyser). What is being described is defining multiplication of cardinals by taking essentially the cardinal of a Cartesian product of "factors" (sets), but using sets (classes) rather than tuples to form the product. For a set containing exactly one element from each factor to faithfully represent an element of the Cartesian product of the factors, it is essential that no two factors have an element in common. So: the factors must be pairwise disjoint.
        $endgroup$
        – Marc van Leeuwen
        Mar 11 at 11:04











      • $begingroup$
        ... What I just said is not quite exact: if two or more factors share an element, then the problem is not so much that a set containing exactly one element from each factor fails to identify an element of their Cartesian product, the problem is that some elements of the Cartesian product (i.e., some tuples) cannot be represented by such a set. This is because the set of components of the tuple would have more than one element in common with certain factors, so fail the requirement. But it remains: the factors need to be pairwise disjoint for the construction to define cardinal multiplication.
        $endgroup$
        – Marc van Leeuwen
        Mar 11 at 11:14










      • $begingroup$
        For concreteness, $k=0,0,1,1$ is not a multiplicative class, though $bigcap k =emptyset$. Indeed the "class of all the classes each of which contains one and but one term of each class in $k$" is empty, as one easily sees; however the cardinal multiplication should result in a class with $1times2times1=2$ members. For a disjoint class of classes, one would get the right cardinal for the multiplicative class. Final remark: maybe only multiplication of $2$ cardinals is intended here; then of course "disjoint" only applies to two sets and one gets no verdict at all from it
        $endgroup$
        – Marc van Leeuwen
        Mar 11 at 11:36











      • $begingroup$
        @MarcvanLeeuwen I agree - thanks for the important correction. It definitely means pairwise in the first quote. I edited the answer.
        $endgroup$
        – echinodermata
        Mar 15 at 5:48












      • 1




        $begingroup$
        Thanks for the great research work. I do not agree with your conclusion about the first citation ("The Thesis of Modern Logistic" by C. J. Keyser). What is being described is defining multiplication of cardinals by taking essentially the cardinal of a Cartesian product of "factors" (sets), but using sets (classes) rather than tuples to form the product. For a set containing exactly one element from each factor to faithfully represent an element of the Cartesian product of the factors, it is essential that no two factors have an element in common. So: the factors must be pairwise disjoint.
        $endgroup$
        – Marc van Leeuwen
        Mar 11 at 11:04











      • $begingroup$
        ... What I just said is not quite exact: if two or more factors share an element, then the problem is not so much that a set containing exactly one element from each factor fails to identify an element of their Cartesian product, the problem is that some elements of the Cartesian product (i.e., some tuples) cannot be represented by such a set. This is because the set of components of the tuple would have more than one element in common with certain factors, so fail the requirement. But it remains: the factors need to be pairwise disjoint for the construction to define cardinal multiplication.
        $endgroup$
        – Marc van Leeuwen
        Mar 11 at 11:14










      • $begingroup$
        For concreteness, $k=0,0,1,1$ is not a multiplicative class, though $bigcap k =emptyset$. Indeed the "class of all the classes each of which contains one and but one term of each class in $k$" is empty, as one easily sees; however the cardinal multiplication should result in a class with $1times2times1=2$ members. For a disjoint class of classes, one would get the right cardinal for the multiplicative class. Final remark: maybe only multiplication of $2$ cardinals is intended here; then of course "disjoint" only applies to two sets and one gets no verdict at all from it
        $endgroup$
        – Marc van Leeuwen
        Mar 11 at 11:36











      • $begingroup$
        @MarcvanLeeuwen I agree - thanks for the important correction. It definitely means pairwise in the first quote. I edited the answer.
        $endgroup$
        – echinodermata
        Mar 15 at 5:48







      1




      1




      $begingroup$
      Thanks for the great research work. I do not agree with your conclusion about the first citation ("The Thesis of Modern Logistic" by C. J. Keyser). What is being described is defining multiplication of cardinals by taking essentially the cardinal of a Cartesian product of "factors" (sets), but using sets (classes) rather than tuples to form the product. For a set containing exactly one element from each factor to faithfully represent an element of the Cartesian product of the factors, it is essential that no two factors have an element in common. So: the factors must be pairwise disjoint.
      $endgroup$
      – Marc van Leeuwen
      Mar 11 at 11:04





      $begingroup$
      Thanks for the great research work. I do not agree with your conclusion about the first citation ("The Thesis of Modern Logistic" by C. J. Keyser). What is being described is defining multiplication of cardinals by taking essentially the cardinal of a Cartesian product of "factors" (sets), but using sets (classes) rather than tuples to form the product. For a set containing exactly one element from each factor to faithfully represent an element of the Cartesian product of the factors, it is essential that no two factors have an element in common. So: the factors must be pairwise disjoint.
      $endgroup$
      – Marc van Leeuwen
      Mar 11 at 11:04













      $begingroup$
      ... What I just said is not quite exact: if two or more factors share an element, then the problem is not so much that a set containing exactly one element from each factor fails to identify an element of their Cartesian product, the problem is that some elements of the Cartesian product (i.e., some tuples) cannot be represented by such a set. This is because the set of components of the tuple would have more than one element in common with certain factors, so fail the requirement. But it remains: the factors need to be pairwise disjoint for the construction to define cardinal multiplication.
      $endgroup$
      – Marc van Leeuwen
      Mar 11 at 11:14




      $begingroup$
      ... What I just said is not quite exact: if two or more factors share an element, then the problem is not so much that a set containing exactly one element from each factor fails to identify an element of their Cartesian product, the problem is that some elements of the Cartesian product (i.e., some tuples) cannot be represented by such a set. This is because the set of components of the tuple would have more than one element in common with certain factors, so fail the requirement. But it remains: the factors need to be pairwise disjoint for the construction to define cardinal multiplication.
      $endgroup$
      – Marc van Leeuwen
      Mar 11 at 11:14












      $begingroup$
      For concreteness, $k=0,0,1,1$ is not a multiplicative class, though $bigcap k =emptyset$. Indeed the "class of all the classes each of which contains one and but one term of each class in $k$" is empty, as one easily sees; however the cardinal multiplication should result in a class with $1times2times1=2$ members. For a disjoint class of classes, one would get the right cardinal for the multiplicative class. Final remark: maybe only multiplication of $2$ cardinals is intended here; then of course "disjoint" only applies to two sets and one gets no verdict at all from it
      $endgroup$
      – Marc van Leeuwen
      Mar 11 at 11:36





      $begingroup$
      For concreteness, $k=0,0,1,1$ is not a multiplicative class, though $bigcap k =emptyset$. Indeed the "class of all the classes each of which contains one and but one term of each class in $k$" is empty, as one easily sees; however the cardinal multiplication should result in a class with $1times2times1=2$ members. For a disjoint class of classes, one would get the right cardinal for the multiplicative class. Final remark: maybe only multiplication of $2$ cardinals is intended here; then of course "disjoint" only applies to two sets and one gets no verdict at all from it
      $endgroup$
      – Marc van Leeuwen
      Mar 11 at 11:36













      $begingroup$
      @MarcvanLeeuwen I agree - thanks for the important correction. It definitely means pairwise in the first quote. I edited the answer.
      $endgroup$
      – echinodermata
      Mar 15 at 5:48




      $begingroup$
      @MarcvanLeeuwen I agree - thanks for the important correction. It definitely means pairwise in the first quote. I edited the answer.
      $endgroup$
      – echinodermata
      Mar 15 at 5:48











      7












      $begingroup$

      In this context disjoint means $A cap B cap C = emptyset$.






      share|cite|improve this answer









      $endgroup$








      • 12




        $begingroup$
        Is that a standard meaning? I never saw the term formally defined that way in four years of undergrad math classes.
        $endgroup$
        – Connor Harris
        Mar 8 at 15:52






      • 1




        $begingroup$
        me neither, but four people have answered the question this way in four minutes!
        $endgroup$
        – John Lawrence Aspden
        Mar 8 at 15:57






      • 2




        $begingroup$
        If you define "disjoint" to mean "empty intersection" (which is the standard definition) then formally for a family of sets "disjoint" would mean the intersection of the entire family is empty unless stated otherwise. The use of the pleonastic term "pairwise" helps to avoid confusion.
        $endgroup$
        – Umberto P.
        Mar 8 at 16:06







      • 1




        $begingroup$
        This is not the standard definition of disjoint. Google it if you like.
        $endgroup$
        – Marc van Leeuwen
        Mar 8 at 19:47






      • 5




        $begingroup$
        @Brilliand Indeed, somebody trying to find evidence for the fact that disjoint means empty intersection might land on the current page and find plenty of evidence to support it. But if you bluntly proclaim as this answer does that this is what the term means than it does not hurt to check that you can find evidence of a text using it that way, and in spite of the multitude of answers here in the same sense, nobody has been able to do that.
        $endgroup$
        – Marc van Leeuwen
        Mar 9 at 5:54















      7












      $begingroup$

      In this context disjoint means $A cap B cap C = emptyset$.






      share|cite|improve this answer









      $endgroup$








      • 12




        $begingroup$
        Is that a standard meaning? I never saw the term formally defined that way in four years of undergrad math classes.
        $endgroup$
        – Connor Harris
        Mar 8 at 15:52






      • 1




        $begingroup$
        me neither, but four people have answered the question this way in four minutes!
        $endgroup$
        – John Lawrence Aspden
        Mar 8 at 15:57






      • 2




        $begingroup$
        If you define "disjoint" to mean "empty intersection" (which is the standard definition) then formally for a family of sets "disjoint" would mean the intersection of the entire family is empty unless stated otherwise. The use of the pleonastic term "pairwise" helps to avoid confusion.
        $endgroup$
        – Umberto P.
        Mar 8 at 16:06







      • 1




        $begingroup$
        This is not the standard definition of disjoint. Google it if you like.
        $endgroup$
        – Marc van Leeuwen
        Mar 8 at 19:47






      • 5




        $begingroup$
        @Brilliand Indeed, somebody trying to find evidence for the fact that disjoint means empty intersection might land on the current page and find plenty of evidence to support it. But if you bluntly proclaim as this answer does that this is what the term means than it does not hurt to check that you can find evidence of a text using it that way, and in spite of the multitude of answers here in the same sense, nobody has been able to do that.
        $endgroup$
        – Marc van Leeuwen
        Mar 9 at 5:54













      7












      7








      7





      $begingroup$

      In this context disjoint means $A cap B cap C = emptyset$.






      share|cite|improve this answer









      $endgroup$



      In this context disjoint means $A cap B cap C = emptyset$.







      share|cite|improve this answer












      share|cite|improve this answer



      share|cite|improve this answer










      answered Mar 8 at 15:52









      Umberto P.Umberto P.

      40.3k13370




      40.3k13370







      • 12




        $begingroup$
        Is that a standard meaning? I never saw the term formally defined that way in four years of undergrad math classes.
        $endgroup$
        – Connor Harris
        Mar 8 at 15:52






      • 1




        $begingroup$
        me neither, but four people have answered the question this way in four minutes!
        $endgroup$
        – John Lawrence Aspden
        Mar 8 at 15:57






      • 2




        $begingroup$
        If you define "disjoint" to mean "empty intersection" (which is the standard definition) then formally for a family of sets "disjoint" would mean the intersection of the entire family is empty unless stated otherwise. The use of the pleonastic term "pairwise" helps to avoid confusion.
        $endgroup$
        – Umberto P.
        Mar 8 at 16:06







      • 1




        $begingroup$
        This is not the standard definition of disjoint. Google it if you like.
        $endgroup$
        – Marc van Leeuwen
        Mar 8 at 19:47






      • 5




        $begingroup$
        @Brilliand Indeed, somebody trying to find evidence for the fact that disjoint means empty intersection might land on the current page and find plenty of evidence to support it. But if you bluntly proclaim as this answer does that this is what the term means than it does not hurt to check that you can find evidence of a text using it that way, and in spite of the multitude of answers here in the same sense, nobody has been able to do that.
        $endgroup$
        – Marc van Leeuwen
        Mar 9 at 5:54












      • 12




        $begingroup$
        Is that a standard meaning? I never saw the term formally defined that way in four years of undergrad math classes.
        $endgroup$
        – Connor Harris
        Mar 8 at 15:52






      • 1




        $begingroup$
        me neither, but four people have answered the question this way in four minutes!
        $endgroup$
        – John Lawrence Aspden
        Mar 8 at 15:57






      • 2




        $begingroup$
        If you define "disjoint" to mean "empty intersection" (which is the standard definition) then formally for a family of sets "disjoint" would mean the intersection of the entire family is empty unless stated otherwise. The use of the pleonastic term "pairwise" helps to avoid confusion.
        $endgroup$
        – Umberto P.
        Mar 8 at 16:06







      • 1




        $begingroup$
        This is not the standard definition of disjoint. Google it if you like.
        $endgroup$
        – Marc van Leeuwen
        Mar 8 at 19:47






      • 5




        $begingroup$
        @Brilliand Indeed, somebody trying to find evidence for the fact that disjoint means empty intersection might land on the current page and find plenty of evidence to support it. But if you bluntly proclaim as this answer does that this is what the term means than it does not hurt to check that you can find evidence of a text using it that way, and in spite of the multitude of answers here in the same sense, nobody has been able to do that.
        $endgroup$
        – Marc van Leeuwen
        Mar 9 at 5:54







      12




      12




      $begingroup$
      Is that a standard meaning? I never saw the term formally defined that way in four years of undergrad math classes.
      $endgroup$
      – Connor Harris
      Mar 8 at 15:52




      $begingroup$
      Is that a standard meaning? I never saw the term formally defined that way in four years of undergrad math classes.
      $endgroup$
      – Connor Harris
      Mar 8 at 15:52




      1




      1




      $begingroup$
      me neither, but four people have answered the question this way in four minutes!
      $endgroup$
      – John Lawrence Aspden
      Mar 8 at 15:57




      $begingroup$
      me neither, but four people have answered the question this way in four minutes!
      $endgroup$
      – John Lawrence Aspden
      Mar 8 at 15:57




      2




      2




      $begingroup$
      If you define "disjoint" to mean "empty intersection" (which is the standard definition) then formally for a family of sets "disjoint" would mean the intersection of the entire family is empty unless stated otherwise. The use of the pleonastic term "pairwise" helps to avoid confusion.
      $endgroup$
      – Umberto P.
      Mar 8 at 16:06





      $begingroup$
      If you define "disjoint" to mean "empty intersection" (which is the standard definition) then formally for a family of sets "disjoint" would mean the intersection of the entire family is empty unless stated otherwise. The use of the pleonastic term "pairwise" helps to avoid confusion.
      $endgroup$
      – Umberto P.
      Mar 8 at 16:06





      1




      1




      $begingroup$
      This is not the standard definition of disjoint. Google it if you like.
      $endgroup$
      – Marc van Leeuwen
      Mar 8 at 19:47




      $begingroup$
      This is not the standard definition of disjoint. Google it if you like.
      $endgroup$
      – Marc van Leeuwen
      Mar 8 at 19:47




      5




      5




      $begingroup$
      @Brilliand Indeed, somebody trying to find evidence for the fact that disjoint means empty intersection might land on the current page and find plenty of evidence to support it. But if you bluntly proclaim as this answer does that this is what the term means than it does not hurt to check that you can find evidence of a text using it that way, and in spite of the multitude of answers here in the same sense, nobody has been able to do that.
      $endgroup$
      – Marc van Leeuwen
      Mar 9 at 5:54




      $begingroup$
      @Brilliand Indeed, somebody trying to find evidence for the fact that disjoint means empty intersection might land on the current page and find plenty of evidence to support it. But if you bluntly proclaim as this answer does that this is what the term means than it does not hurt to check that you can find evidence of a text using it that way, and in spite of the multitude of answers here in the same sense, nobody has been able to do that.
      $endgroup$
      – Marc van Leeuwen
      Mar 9 at 5:54











      4












      $begingroup$

      Consider the sets $A = 1,2$, $B = 2,3$, $C = 3, 1$. Then $Acap Bcap C = varnothing$, but $A,B,C$ are not pairwise disjoint.






      share|cite|improve this answer









      $endgroup$

















        4












        $begingroup$

        Consider the sets $A = 1,2$, $B = 2,3$, $C = 3, 1$. Then $Acap Bcap C = varnothing$, but $A,B,C$ are not pairwise disjoint.






        share|cite|improve this answer









        $endgroup$















          4












          4








          4





          $begingroup$

          Consider the sets $A = 1,2$, $B = 2,3$, $C = 3, 1$. Then $Acap Bcap C = varnothing$, but $A,B,C$ are not pairwise disjoint.






          share|cite|improve this answer









          $endgroup$



          Consider the sets $A = 1,2$, $B = 2,3$, $C = 3, 1$. Then $Acap Bcap C = varnothing$, but $A,B,C$ are not pairwise disjoint.







          share|cite|improve this answer












          share|cite|improve this answer



          share|cite|improve this answer










          answered Mar 8 at 15:53









          Kyle DuffyKyle Duffy

          843




          843





















              0












              $begingroup$

              If disjoint is understood as having an empty intersection, then disjoint sets are not necessarily pairwise disjoint. Pairwise disjoint avoid any doubt, at no cost.






              share|cite|improve this answer









              $endgroup$

















                0












                $begingroup$

                If disjoint is understood as having an empty intersection, then disjoint sets are not necessarily pairwise disjoint. Pairwise disjoint avoid any doubt, at no cost.






                share|cite|improve this answer









                $endgroup$















                  0












                  0








                  0





                  $begingroup$

                  If disjoint is understood as having an empty intersection, then disjoint sets are not necessarily pairwise disjoint. Pairwise disjoint avoid any doubt, at no cost.






                  share|cite|improve this answer









                  $endgroup$



                  If disjoint is understood as having an empty intersection, then disjoint sets are not necessarily pairwise disjoint. Pairwise disjoint avoid any doubt, at no cost.







                  share|cite|improve this answer












                  share|cite|improve this answer



                  share|cite|improve this answer










                  answered Mar 14 at 17:19









                  Yves DaoustYves Daoust

                  132k676230




                  132k676230



























                      draft saved

                      draft discarded
















































                      Thanks for contributing an answer to Mathematics Stack Exchange!


                      • Please be sure to answer the question. Provide details and share your research!

                      But avoid


                      • Asking for help, clarification, or responding to other answers.

                      • Making statements based on opinion; back them up with references or personal experience.

                      Use MathJax to format equations. MathJax reference.


                      To learn more, see our tips on writing great answers.




                      draft saved


                      draft discarded














                      StackExchange.ready(
                      function ()
                      StackExchange.openid.initPostLogin('.new-post-login', 'https%3a%2f%2fmath.stackexchange.com%2fquestions%2f3140228%2fwhy-do-we-say-pairwise-disjoint-rather-than-disjoint%23new-answer', 'question_page');

                      );

                      Post as a guest















                      Required, but never shown





















































                      Required, but never shown














                      Required, but never shown












                      Required, but never shown







                      Required, but never shown

































                      Required, but never shown














                      Required, but never shown












                      Required, but never shown







                      Required, but never shown






                      Popular posts from this blog

                      How to check contact read email or not when send email to Individual?

                      Bahrain

                      Postfix configuration issue with fips on centos 7; mailgun relay